Tuesday 13 November 2012

Oncology and Hematology multiple choice questions 3

 
Q 1 . A 22-year-old pregnant woman of northern European
descent presents 3 months into her first pregnancy
with extreme fatigue, pallor, and icterus. She reports being
previously healthy. On evaluation her hemoglobin is 8
g/dL, reticulocyte count is 9%, indirect bilirubin is 4.9
mg/dL, and serum haptoglobin is not detectable. Her
physical examination is notable for splenomegaly and a
normal 3-month uterus. Peripheral smear is shown below .
What is the most likely diagnosis? (See Figure III-21,
Color Atlas.)
A. Colonic polyp
B. G6PD deficiency
C. Hereditary spherocytosis
D. Parvovirus B19 infection
E. Thrombotic thrombocytopenic purpura


Answer   C



Q 2 . A patient with acute lymphoid leukemia (ALL) is
admitted with respiratory distress and chest pain. The patient
reports 1 day of shortness of breath not associated
with cough. There have been no sick contacts, and before
the onset of the respiratory symptoms, the patient only
recalls fatigue. A chest radiograph shows faint diffuse interstitial
infiltrates without pulmonary edema. The cardiac
silhouette is normal. An arterial blood gas shows a
PaO2 = 54 mmHg, while the pulse oximetry is 97% on
room air. A carbon monoxide level is normal. All of the
following laboratory abnormalities are expected in this
patient except
A. bcr-abl mutation
B. blast count >100,000/ μL
C. elevated lactate dehydrogenase levels
D. increased blood viscosity
E. methemoglobinemia


Answer   E



Q 3 . A 48-year-old male is referred for evaluation by an
acute care center because of a nodule on chest radiography.
Three weeks ago he was diagnosed with pneumonia after
reporting 3 days of fever, cough, and sputum production.
The chest radiogram showed a small right lower lobe alveolar
infiltrate and a left upper lobe 1.5-cm round nodule.
He was treated with antibiotics and is now asymptomatic.
A repeat chest radiogram shows that the right lower lobe
pneumonia is resolved, but the nodule is still present. He is
asymptomatic. He smoked one pack of cigarettes per day
for 25 years and quit 3 years ago. He never had a prior chest
radiogram. CT scan shows that the nodule is 1.5 by 1.7 cm
and is located centrally in the left upper lobe, has no calcification,
and has slightly scalloped edges. There is no mediastinal
adenopathy or pleural effusion. Which of the
following is the appropriate next step in his management?
A. Bronchoscopy
B. Mediastinoscopy
C. MRI scan
D. 18FDG PET scan
E. Repeat chest CT in 6 months


Answer    D




Q 4 . All the following types of cancer commonly metastasize
to the central nervous system (CNS) except
A. ovarian
B. breast
C. hypernephroma
D. melanoma
E. acute lymphoblastic leukemia (ALL)



Answer    A



Q 5 . A 54-year-old woman with atrial fibrillation is anticoagulated
with warfarin, 5 mg daily. She developed a urinary
tract infection that her primary care physician has treated
with ciprofloxacin, 250 mg orally twice daily for 7 days. She
presents to the emergency room today complaining of
blood in her urine and easy bruising. Her physical examination
shows ecchymoses on her arms. Her urine is bloody
in appearance, but no clots are present. After flushing the
bladder with 100 mL of sterile saline, the urine returns
with a slight pink hue only. A urinalysis shows 3–5 white
blood cells per high power field and many red blood cells
per high power field. There are no bacteria present. The international
normalized ratio (INR) is 7.0. What is the best
approach to treatment of this patient’s coagulopathy?
A. Administer vitamin K 10 mg IV.
B. Administer vitamin K 2 mg SC.
C. Administer vitamin K 1 mg sublingually.
D. Hold further warfarin doses until the INR falls to 2.0.
E. Transfuse four units of fresh-frozen plasma.


Answer    C



Q  6 . Which of the following statements about cardiac
toxicity from cancer treatment is true?
A. Doxorubicin-based cardiac toxicity is idiosyncratic
and dose-independent.
B. Anthracycline-induced congestive heart failure is reversible
with time and control of risk factors.
C. Mediastinal irradiation often results in acute pericarditis
during the first few weeks of treatment.
D. Chronic constrictive pericarditis often manifests
symptomatically up to 10 years after treatment.
E. The incidence of coronary atherosclerosis in patients
who have a history of mediastinal irradiation
is the same as that in age-matched controls.



Answer     D


Q 7 . A 23-year-old woman is diagnosed with a lower extremity
deep venous thrombosis. Which of the following
medical conditions represents a contraindication to therapy
with low-molecular-weight heparin (LMWH)?
A. Pregnancy
B. Obesity
C. Dialysis-dependent renal failure
D. Uncontrolled diabetes mellitus
E. Jaundice


Answer    C


Q 8  . Which of the following pairs of chemotherapy and
complication is incorrect?
A. Daunorubicin—CHF
B. Bleomycin—interstitial fibrosis
C. Cyclophosphamide—hematuria
D. Cisplatin—liver failure
E. Ifosfamide—Fanconi syndrome


Answer    D



Q 9 . A 70-year-old man is admitted to the cardiac care
unit for complaints of chest pressure occurring at rest radiating
to his left arm with associated diaphoresis and presyncope.
His admission electrocardiogram (ECG) showed
ST depressions in V4–V6. The chest pain and ECG
changes resolve with sublingual nitroglycerin. He is
treated with IV heparin, aspirin, metoprolol, and lisinopril.
His cardiac catheterization shows 90% occlusion of
the left anterior descending artery, 80% occlusion of the
distal circumflex artery, and 99% occlusion of the right
coronary artery. He remains in the cardiac care unit awaiting
coronary artery bypass. He has a history of rheumatic
heart disease and underwent mechanical mitral valve replacement
at age 58. On admission, his hemoglobin is 12.2
g/dL, hematocrit 37.1%, white blood cell (WBC) count
9800/μL, and platelet count 240,000/μL. His creatinine is
1.7 mg/dL. On the fourth hospital day, his hemoglobin is
10.0, hematocrit 31%, WBC count 7600/μL, and platelet
count 112,000/μL. His creatinine has risen to 2.9 mg/dL
after the cardiac catheterization. What is the most appropriate
treatment of the patient at this time?
A. Continue heparin and give a platelet transfusion.
B. Discontinue heparin infusion and start argatroban.
C. Discontinue heparin and start lepirudin.
D. Discontinue heparin and start warfarin.
E. Send serum to assess for the presence of heparin–
platelet factor 4 (PF4) IgG antibody and continue
heparin.



Answer  B



Q 10 . A 24-year-old woman presents to the emergency
room complaining of a red, tender rash that has been
spreading across her arms and legs over the past 2 days. She
also describes severe diffuse muscle pain that has worsened
over a week’s time. She woke up feeling as though she could
not catch her breath and has developed a dry cough over
the past several days. She is without any significant medical
history but recalls that she had similar symptoms several
years ago, and was told she was having an allergic reaction.
Her symptoms abated with an oral glucocorticoid taper.
She takes no prescription medications but takes a number
of over-the-counter nutritional supplements daily. She cannot
describe any allergic trigger to her previous episode or
her current one. Her family history is unremarkable, and
her close contacts are not ill. She works in an office, has no
pets, and has not travelled internationally. Her laboratory
results are remarkable for a leukocyte count of 12,100 cells/
μL and a total eosinophil count of 1100/μ L. Which of the
following is the most likely cause of her symptoms?
A. Early stage of systemic lupus erythematosus
B. Gluten allergy
C. Ingestion of L-tryptophan
D. Lactose intolerance
E. Recent viral upper respiratory tract infection



Answer    C


No comments:

Post a Comment